Download as pdf or txt
Download as pdf or txt
You are on page 1of 36

LEGALEDGE TEST SERIES

Part of the Most Comprehensive & Consistently Successful Study Material & Test Series Module, spanning across
both Physical and Online Programs in the entire Country. As a result, LegalEdge was able to engineer Clean-Sweep-
Landslide figures of a handsome 64 Selections under 100 ranks, and a whopping 273 selections under 500 ranks in
CLAT 2021. With AILET being no different, a total of 34 of our students found their way into NLU, Delhi in 2021.
In a nutshell, every second admit in a Top National Law School in 2021 came from the LegalEdge Preparation
Ecosystem.

MOCK COMMON LAW ADMISSION TEST 2022


MOCK CLAT #31

Scan this code after the test


Duration : 120 Minutes Candidate Name : _____________
Max. Marks : 150 Batch : _____________
Centre Name : __________ Contact No. : _____________

to punch in your answers


(Test ID: 2461488)
INSTRUCTIONS TO CANDIDATES

1. No clarification on the question paper can be sought. Answer the questions as they are.
2. There are 150 multiple choice objective type questions.
3. There is negative marking of 0.25 for every incorrect answer. Each question carries ONE mark. Total marks are
150
4. You have to indicate the correct answer by darkening one of the four responses provided, with a BALL PEN
(BLUE OR BLACK) in the OMR Answer Sheet.
Example: For the question, "Where is the Taj Mahal located?", the correct answer is (b).
The student has to darken the corresponding circle as indicated below:
(a) Kolkata (b) Agra (c) Bhopal (d) Delhi
Right Method Wrong Methods

5. Answering the questions by any method other than the method indicated above shall be considered incorrect and
no marks will be awarded for the same.
6. More than one response to a question shall be counted as wrong.
7. Do not write anything on the OMR Answer Sheet other than the details required and, in the spaces, provided for.
8. You are not required to submit the OMR Answer Sheet and Test Paper after the test.
9. The use of any unfair means by any candidate shall result in the cancellation of his/her candidature.
10. Impersonation is an offence and the student, apart from disqualification, may have to face criminal prosecution.
11. You have to scan the QR code only after completion of offline test.
12. You cannot leave the examination hall without punching your answers on the portal.
SECTION-A : ENGLISH LANGUAGE

Directions(Q.1-Q.30): Read the following passage carefully and answer the questions that follow.
Passage (Q.1-Q.5): During the second half of World War II, a soldier named Yossarian is stationed with his Air
Force squadron on the island of Pianosa, near the Italian coast in the Mediterranean Sea. Yossarian and his
friends endure a nightmarish, absurd existence defined by bureaucracy and violence: they are inhuman resources
in the eyes of their blindly ambitious superior officers. The squadron is thrown thoughtlessly into brutal combat
situations and bombing runs in which it is more important for the squadron members to capture good aerial
photographs of explosions than to destroy their targets. Their colonels continually raise the number of missions
that they are required to fly before being sent home, so that no one is ever sent home. Still, no one but Yossarian
seems to realize that there is a war going on; everyone thinks he is crazy when he insists that millions of people
are trying to kill him.

Yossarian’s story forms the core of the novel, so most events are refracted through his point of view. Yossarian
takes the whole war personally: not swayed by national ideals or abstract principles, Yossarian is furious that his
life is in constant danger through no fault of his own. He has a strong desire to live and is determined to be
immortal or die trying. As a result, he spends a great deal of his time in the hospital, faking various illnesses in
order to avoid the war. As the novel progresses through its loosely connected series of recurring stories and
anecdotes, Yossarian is continually troubled by his memory of Snowden, a soldier who died in his arms on a
mission when Yossarian lost all desire to participate in the war. Yossarian is placed in ridiculous, absurd,
desperate, and tragic circumstances—he sees friends die and disappear, his squadron get bombed by its own
mess officer, and colonels and generals volunteer their men for (------) most perilous battle in order to enhance
their own reputations.

Catch-22 is a law defined in various ways throughout the novel. First, Yossarian discovers that it is possible to
be discharged from military service because of insanity. Always looking for a way out, Yossarian claims that he
is insane, only to find out that by claiming that he is insane he has proved that he is obviously sane—since any
sane person would claim that he or she is insane in order to avoid flying bombing missions. Elsewhere, Catch-
22 is defined as a law that is illegal to read. Ironically, the place where it is written that it is illegal is in Catch-
22 itself. It is yet again defined as the law that the enemy is allowed to do anything that one can’t keep him from
doing.
https://www.sparknotes.com/lit/catch22/summary/

1. According to the passage, what is the meaning of Catch-22?


(a) Catch-22 is any paradoxical, circular reasoning that catches its victim in its illogic.
(b) Catch-22 is an endless cycle of logically consistent arguments put forth one after the other.
(c) Catch-22 is a measure or policy whose effect is the opposite of what was intended
(d) Catch-22 is a situation presenting two equally undesirable alternatives.

2. The mission, wherein the death of Snowden occurred, brought a significant change in Yossarian. What was that
change?
(a) He was continually troubled by his memories.
(b) He acquired a strong desire to die beside his friend.
(c) He acquired a strong desire to live and be immortal.
(d) He lost his will to fight the in the war.

3. What is the tone of the author?


(a) Satirical (b) Sceptical (c) Cynical (d) Ironical

4. Who among the following is/are representative of brutish selfishness?


(a) Yossarian. (b) Snowden. (c) Generals. (d) Pianosa.
Head Office: 127, Zone II, MP Nagar, Bhopal |+91-7676564400| https://www.toprankers.com Page 2 of 36
5. Insert the appropriate article in the above italicised line to make it grammatically correct-
Yossarian is placed in ridiculous, absurd, desperate, and tragic circumstances—he sees friends die and
disappear, his squadron get bombed by its own mess officer, and colonels and generals volunteer their men for
(-------) most perilous battle in order to enhance their own reputations.
(a) A (b) An (c) The (d) No insertion required

Passage (Q.6-Q.10): Child psychology is the study of subconscious and conscious childhood development.
Child psychologists observe how a child interacts with their parents, themselves, and the world, to understand
their mental development.
Everyone wants their child to have healthy development, but it’s not always clear if a child’s behaviour is a
symptom of a normal stage in development or a sign of an abnormality. Child psychologists can help you
understand the difference. Understanding the normal and abnormal psychological patterns of a child can help
parents understand how to best communicate and connect with their child, teach their child coping mechanisms
for managing emotions, and help their child progress and thrive in each new developmental stage.
Child psychologists can also identify abnormal behaviours early, help detect the root of common behavioural
issues such as learning issues, hyperactivity, or anxiety, and help children work through early childhood trauma.
They can also help to prevent, evaluate, and diagnose developmental delays or abnormalities such as autism.
Physical development in children is typically a predictable sequence of events. Your child holds their head up,
rolls over, crawls, walks, and runs, in that order. Your Child Psychologist can aid your paediatrician in observing
your child’s physical development, and if there are any abnormalities that could indicate developmental
irregularities. Child Psychologists will observe your child’s progression toward the milestones of development
to ensure that your child is physically developing normally. Major delays in physical development may reveal
other underlying developmental issues that can then be addressed early on.
The medical understanding of childhood cognitive development has greatly changed over the recent years.
Emotional and social development are deeply intertwined. Emotional development refers to how the child feels,
understands, and expresses their emotions. Emotional development is expressed in very young children through
the expression of basic emotions like fear, joy, anger, and sadness. As the child develops, more complex emotions
such as confidence, hope, guilt, and pride emerge. Emotional development also includes a child’s ability to feel
and understand the emotions of other people through empathy. Learning to regulate and express emotions
appropriately is difficult for many children. Helping children understand their emotions early can have a
powerful impact on current and future emotional development. A Child Psychologist can help your child
understand and express their emotions in a healthy, positive way.
Emotional development highly informs social development. Social development is about how the child develops
the values, awareness, and social skills necessary to relate to the people around them; their parents, peers,
authority figures, and animals. A child’s early relationships can have a huge impact on their development of age-
appropriate social skills. Trust, friendship, conflict management, and respect for authority are examples of social
development.

6. Based on the passage, in what way is Child psychology helpful?


(a) Child psychology helps an adult identify abnormal behaviours early, help detect the root of common
behavioural issues such as learning issues, hyperactivity, or anxiety, and help children work through early
childhood trauma.
(b) Child psychology develops a child’s social values, awareness and social skills necessary to relate to the
people around them
(c) Child psychology helps in observing a child’s interaction with their parents, themselves, and the world by
understanding their cognitive, emotional and social development.
(d) It makes the parent more conscious about the abnormality of their child.

Head Office: 127, Zone II, MP Nagar, Bhopal |+91-7676564400| https://www.toprankers.com Page 3 of 36
7. According to the passage, which among the following is not true?
(a) Identifying the abnormality in the early stage can solve problems like autism.
(b) Psychologists study the activity of the child and can identify if there is any abnormality in the behaviour.
(c) Learning issues, hyperactivity, or anxiety are issues that are dealt by the Child psychologist.
(d) The medical understanding of childhood cognitive development has advanced in transforming a child’s
mental make-up.

8. ‘Emotional development highly informs social development.’ Which of the following is the most suitable
interpretation of the statement in isolation?
(a) Emotional and social developments are parallel to each other.
(b) Emotional and social developments are interwoven.
(c) Social development is not really important at an early age.
(d) Emotional development indicates social development.

9. Which of the following is a suitable title for the passage.?


(a) Definition of Child Psychology.
(b) Scope of Child Psychology.
(c) Introduction to Child Psychology.
(d) Child’s Psychology.

10. What is the main function of psychologists while working with children?
(a) To point out the disabilities of the child and provide proper medicine.
(b) To identify the root cause of the psychological, cognitive and social problems of a child.
(c) To guide the parents to communicate in the right way with the children.
(d) To collate the data of behavioural aspects of children for progressive analysis.

Passage (Q.11-Q.15): Many people seem older than they are. Research into the causes of premature aging has
shown that stress has a lot to do with it, because the body wears down much faster during periods of crisis. The
American Institute of Stress investigated this degenerative process and concluded that most health problems are
caused by stress. Researchers at the Heidelberg University Hospital conducted a study in which they subjected
a young doctor to a job interview, which they made even more stressful by forcing him to solve complex math
problems for thirty minutes. Afterward, they took a blood sample. What they discovered was that his antibodies
had reacted to stress the same way they react to pathogens, activating the proteins that trigger an immune
response. The problem is that this response not only neutralizes harmful agents, it also damages healthy cells,
leading them to age prematurely. The University of California conducted a similar study, taking data and samples
from thirty-nine women who had high levels of stress due to the illness of one of their children and comparing
them to samples from women with healthy children and low levels of stress. They found that stress promotes
cellular aging by weakening cell structures known as telomeres, which affect cellular regeneration and how our
cells age. As the study revealed, the greater the stress, the greater the degenerative effect on cells.
These days, people live at a frantic pace and in a nearly constant state of competition. At this fever pitch, stress
is a natural response to the information being received by the body as potentially dangerous or problematic.
Theoretically, this is a useful reaction, as it helps us survive in hostile surroundings. Over the course of our
evolution, we have used this response to surroundings. Over the course of our evolution, we have used this
response to deal with difficult situations and to flee from predators. The alarm that goes off in our head makes
our neurons activate the pituitary gland, which produces hormones that release corticotropin, which in turn
circulates through the body via the sympathetic nervous system. Nonetheless, the stress to which human beings
are subjected today is clearly harmful. Stress has a degenerative effect over time. A sustained state of emergency
affects the neurons associated with memory, as well as inhibiting the release of certain hormones, the absence
of which can cause depression. Its secondary effects include irritability, insomnia, anxiety, and high blood

Head Office: 127, Zone II, MP Nagar, Bhopal |+91-7676564400| https://www.toprankers.com Page 4 of 36
pressure. As such, though challenges are good for keeping mind and body active, we should adjust our high-
stress lifestyles in order to avoid the premature aging of our bodies.

11. Which of the following reflects the main objective of the passage?
(a) To make aware the readers the path to a healthy life by adjusting high-stress situations to avoid the premature
aging of our bodies.
(b) To help readers combat high stress lifestyles in order to avoid premature aging of our bodies.
(c) To Highlight the factors leading to premature aging of our body.
(d) To clear misconception surrounding stress, especially when stress is important for survival.

12. With reference to the passage, what does the expression “frantic pace” mean?
(a) Peaceful activities.
(b) Anxiety-driven activities.
(c) Overzealous activities.
(d) Leisurely activities.

13. “Challenges are good to keep mind and body active”. Which among the following supports the statement?
(a) Challenges increase the ability to fight and sometimes lead to a difficult path in life.
(b) Challenges make one sick and hyper leading to more stress.
(c) The change that comes along challenges motivates people and increases their capacity to work optimally.
(d) To some extent, challenges help to keep the mind and body nimble, and this helps us to survive or evolve
with the situation.

14. From the passage, it can be inferred that the author is


I. A reporter.
II. A researcher.
III. A biologist.
IV. A scientist.
(a) I & II (b) II & IV (c) Only III (d) Either II or IV

15. What is the tone of the author in the passage?


(a) Quixotic (b) Pedestrian (c) Analytical (d) Commiserating

Passage (Q.16-Q.20): The COVID-19 pandemic has triggered new ways of learning. All around the world,
educational institutions are looking toward online learning platforms to continue with the process of educating
students. The new normal now is a transformed concept of education with online learning at the core of this
transformation. Today, digital learning has emerged as a necessary resource for students and schools all over the
world. For many educational institutes, this is an entirely new way of education that they have had to adopt.
Online learning is now applicable not just to learn academics but it also extends to learning extracurricular
activities for students as well. In recent months, the demand for online learning has risen significantly, and it
will continue doing so in the future.
For many students, one of the biggest challenges of online learning is the struggle with focusing on the screen
for long periods of time. With online learning, there is also a greater chance for students to be easily distracted
by social media or other sites. Therefore, it is imperative for the teachers to keep their online classes crisp,
engaging, and interactive to help students stay focused on the lesson.
Another key challenge of online classes is internet connectivity. While internet penetration has grown in leaps
and bounds over the past few years, in smaller cities and towns, a consistent connection with decent speed is a
problem. Without a consistent internet connection for students or teachers, there can be a lack of continuity in
learning for the child. This is detrimental to the education process.
Students can learn a lot from being in the company of their peers. However, in an online class, there are minimal
physical interactions between students and teachers. This often results in a sense of isolation for the students. In
Head Office: 127, Zone II, MP Nagar, Bhopal |+91-7676564400| https://www.toprankers.com Page 5 of 36
this situation, it is imperative that the school allow for other forms of communication between the students, peers,
and teachers. This can include online messages, emails and video conferencing that will allow for face-to-face
interaction and reduce the sense of isolation.
Online learning requires teachers to have a basic understanding of using digital forms of learning. However, this
is not the case always. Very often, teachers have a very basic understanding of technology. Sometimes, they
don’t even have the necessary resources and tools to conducts online classes. To combat this, it is important for
schools to invest in training teachers with the latest technology updates so that they can conduct their online
classes seamlessly.
Many parents are concerned about the health hazards of having their children spend so many hours staring at a
screen. This increase in screen time is one of the biggest concerns and disadvantages of online learning.
Sometimes students also develop bad posture and other physical problems due to staying hunched in front of a
screen.

16. Which of the following can be concluded from the passage?


(a) Online education is the newest way of learning after COVID-19
(b) Online education is the only way left for uplifting the education system
(c) Online education has various disadvantages that has to be gradually sorted.
(d) Both A and C

17. Which of the following gives the most appropriate meaning of the word, hunched?
(a) A supine position that causes bending of the spinal curvature.
(b) To squat in a manner that one’s body is bent backwards.
(c) A position where the back is humped in a convex because of abnormal spinal curvature
(d) A position where the spinal curvature is straight, and the neck is slightly bent.

18. Which among the following is not the disadvantage of online education?
(a) Problem with internet connectivity.
(b) Less efficiency in teaching skills.
(c) Easily accessible from every part of the world.
(d) Health hazards for children.

19. “Online education is causing a major feeling of isolation” what does this refer to?
(a) people are getting more open towards education.
(b) Students feel estranged from the company of their peers.
(c) Online education has lessened the social interaction of students.
(d) One of the advantages of online education.

20. What can be the best title for the passage?


(a) Tread cautiously with online education.
(b) Do away with the old, get on with the new.
(c) Online education: a new fad.
(d) Online education: Teachers’ nightmare.

Head Office: 127, Zone II, MP Nagar, Bhopal |+91-7676564400| https://www.toprankers.com Page 6 of 36
Passage (Q.21-Q.25): Soon after World War II, Winston Churchill was visiting the White House when he is
said to have had an uncanny experience. Having had a long bath with a Scotch and cigar, he reportedly walked
into the adjoining bedroom – only to be met by the ghost of Abraham Lincoln. Unflappable, even while
completely naked, Churchill apparently announced: “Good evening, Mr President. You seem to have me at a
disadvantage.” The spirit smiled and vanished.

His supposed contact with the supernatural puts Churchill in illustrious company. Arthur Conan Doyle spoke to
ghosts through mediums, while Alan Turing believed in telepathy. Three men who were all known for their
razor-sharp thinking, yet couldn’t stop themselves from believing in the impossible. You may well join them.
According to recent surveys, as many as three quarters of Americans believe in the paranormal, in some form,
while nearly one in five claim to have actually seen a ghost.

Intrigued by these persistent beliefs, psychologists have started to look at why some of us can’t shake off old
superstitions and folk-lore. Their findings may suggest some hidden virtues to believing in the paranormal. At
the very least, it should cause you to question whether you hold more insidious beliefs about the world.

Some paranormal experiences are easily explainable, based on faulty activity in the brain. Reports of poltergeists
invisibly moving objects seem to be consistent with damage to certain regions of the right hemisphere that are
responsible for visual processing; certain forms of epilepsy, meanwhile, can cause the spooky feeling that a
presence is stalking you close by – perhaps underlying accounts of faceless “shadow people” lurking in the
surroundings.

Out-of-body experiences, meanwhile, are now accepted neurological phenomena, while certain visual illusions
could confound the healthy brain and create mythical beings. For example, one young Italian psychologist looked
in the mirror one morning to find a grizzled old man staring back at him. His later experiments confirmed that
the illusion is surprisingly common when you look at your reflection in the half light, perhaps because the brain
struggles to construct the contours of your face, so it begins to try to fill in the missing information – even if that
leads to the appearance of skulls, old hags or hideous animals.
https://www.bbc.com/future/article/20141030-the-truth-about-the-paranormal

21. What is the main reason behind the occurrence of a paranormal experience?
(a) Aberrant activity in brain.
(b) Neurological occurrence.
(c) Illusions created by the eyes.
(d) People’s firm beliefs in old folklores.

22. According to the passage, which of the following men has had a visual illusion?
(a) Alan Turing (b) Abraham Lincoln
(c) Winston Churchill (d) Arthur Doyle

23. After reading the passage, determine the belief of author in the existence supernatural elements?
(a) Firm believer. He believes in them and regards them as part of real world.
(b) Sceptic. He is unclear towards his stand.
(c) Rejects this idea.
(d) Cannot be determined.

24. ‘Some paranormal experiences are easily explainable, based on faulty activity in the brain.’ Identify the part of
speech of the above underlined word-
(a) Noun (b) Verb (c) Adverb (d) Adjective

Head Office: 127, Zone II, MP Nagar, Bhopal |+91-7676564400| https://www.toprankers.com Page 7 of 36
25. Unflappable, even while completely naked, Churchill apparently announced: “Good evening, Mr President. You
seem to have me at a disadvantage.” The spirit smiled and vanished. Give the synonym of the word in bold –
(a) Befuddled. (b) Imperturbable. (c) Regaled. (d) Disconcerted.

Passage (Q.26-Q.30): Inside our body can be found the bloodiest of battlefields, where millions of organisms
are massacred daily, without cease. It is a battle waged by our body's robust immune system against a wide
variety of pathogenic bacteria, virus, fungi, and parasites. What makes the defence mechanism powerful is the
two-level protection conferred by the immune system. The innate immune system that serves as the first line of
defence is not antigen-specific; it readily targets all pathogenic organisms the moment they enter the body. The
antigen-specific adaptive immune mechanism acts as the second line of protection to keep us healthy. This year's
Nobel Prize in Physiology or Medicine has been awarded to Bruce A. Beutler, Jules A. Hoffmann, and Ralph
M. Steinman for revolutionising our understanding of the immune system by discovering the key principles that
activate the defence mechanism. Beutler and Hoffmann will share half the prize money for discovering the
receptor proteins that recognise micro-organisms and activate the innate immunity. In 1996, Hoffmann found
that the Toll gene was responsible for sensing pathogenic micro-organisms and that its activation was required
for mounting innate immune response. Two years later, Beutler discovered that components of micro-organisms
bind to Toll-like receptors located on many cells. The binding activates the innate immunity, which results in
inflammation and destruction of the pathogens.

The other half of the prize money was awarded to Steinmann for discovering, way back in the 1970s, that
dendritic cells were responsible for adaptive immunity. As they are antigen-specific, dendritic cells take time
to react to an invading organism on first exposure; but immunological memory allows them to react more
rapidly to the same antigen on subsequent exposures. This is the attribute researchers exploit while designing
preventive vaccines. Adaptive immunity holds great medical promise. The immune system can be directed to
attack the tumour. Blocking the excessive production of cytokines when diseases show up can ameliorate
autoimmunity. Even preventing autoimmune diseases may become possible when certain cells of the immune
system are successfully silenced. Steinmann will go down in history as not just a highly worthy Nobel Prize
winner. He was (as a Rockefeller University statement explains) "diagnosed with pancreatic cancer four years
ago ... his life was extended using a dendritic-cell based immunotherapy of his own design," and he died three
days before his Nobel was announced.

26. The primary objective of the passage is


(a) To explain the reason behind the success of Defence mechanism in the body.
(b) To expatiate that the antigen specific immune system acting as a second line of protection works better as
a defence mechanism than innate immune system acting as a first line of defence mechanism
(c) To bring to fore the injustice meted out to the Nobel Prize winners, Bruce A. Beutler and Jules A
Hoffman, by attenuating their works and giving more emphasis to Ralph M Steinman.
(d) To highlight the works of the Nobel Prize winners (in their field) who gave a new direction to our
understanding, by discovering the key principle that activate the defence mechanism and where one
scientist corroborated his theory by applying it on himself.

27. The statement which is not in concord with the text is,
(a) Ralph M. Hoffman found that the Toll gene was responsible for sensing pathogenic micro-organism and
that it was required for mounting innate immune response.
(b) Steinman was awarded the Nobel Prize posthumously
(c) The innate immune system that serves as the first line of defence is not antigen- specific
(d) Steinman, in 1970, discovered that dendritic cell was responsible for adaptive immunity.

28. "Even preventing autoimmune……..are successfully silenced". The word silenced as used in the passage, means
(a) Dead (b) Dormant (c) Resurged (d) Suppressed.

Head Office: 127, Zone II, MP Nagar, Bhopal |+91-7676564400| https://www.toprankers.com Page 8 of 36
29. According to the passage, Dendritic cells are
(a) Pathogen-specific (b) Antigen-specific
(c) Autoimmune (d) Parasites

30. The tone in the passage, is;


(a) Laudatory (b) Belligerent (c) Neutral (d) Satirical

Head Office: 127, Zone II, MP Nagar, Bhopal |+91-7676564400| https://www.toprankers.com Page 9 of 36
SECTION-B : CURRENT AFFAIRS, INCLUDING GENERAL KNOWLEDGE

Directions (Q.31–Q.65): Read the information given below and answer the questions based on it.
Passage (Q.31-Q.35): Western diplomats have set a deadline of later this month to revive the 2015 Iran nuclear
deal, signed under President [1], and dismantled in [2] by President Donald Trump.

American officials are concerned about Iran’s rapid nuclear escalation after the US abandoned the deal, and
President Joe Biden has said that the US will rejoin the agreement if Iran complies with the terms of the original
deal, and if it addresses other issues related to alleged ballistic missile stockpiles and the proxy conflicts that it
backs across the region.

Iran has rejected any deadline imposed by the West, and President Ebrahim Raisi, who is considered a hardliner,
has said that “regional and missile issues are non-negotiable”.
https://indianexpress.com/article/explained/revival-of-iran-nuclear-deal-to-be-geopolitical-considerations-
7755981/

31. Which of the following will replace [1] in the above passage?
(a) Barack Obama (b) George Bush (c) Bill Clinton (d) Ronald Regan

32. Which of the following is TRUE about the Joint Comprehensive Plan of Action?
(a) The deal is a landmark accord signed between Iran and a coalition of world powers collectively known as
P5+1.
(b) Under the deal, Iran agreed to curb its nuclear activity in return for the lifting of sanctions and access to
global trade.
(c) The agreement allowed Iran to accumulate small amounts of uranium for research but it banned the
enrichment of uranium, which is used to make reactor fuel and nuclear weapons.
(d) All of the above

33. Iran’s atomic agency recently, stated that they have enriched 210 kgs of Uranium upto how much percent?
(a) 10% (b) 20% (c) 30% (d) 40%

34. Which of the following is NOT TRUE about International Atomic Energy Agency?
(a) The agency was established by representatives of more than 80 countries in October 1956.
(b) The IAEA’s statute officially came into force on July 29, 1957.
(c) The IAEA and its director general, Mohamed ElBaradei, won the Nobel Prize for Peace in 2015.
(d) It is headquartered in Vienna.

35. Which of the following will replace [2] in the above passage?
(a) 2016 (b) 2017 (c) 2018 (d) 2020

Passage (Q.36-Q.40): Pakistan on Friday signed a new agreement with China to begin the second phase of the
USD 60 billion China-Pakistan Economic Corridor (CPEC) as Prime Minister Imran Khan showered praise on
the controversial projects, saying it has strategic significance for both the countries and delivering tangible
benefits to the people.

Khan, who arrived in China on Thursday on a four-day visit to attend the opening ceremony of the 2020 Beijing
Winter Olympics and meet the top Chinese leadership, held a virtual meeting with He Lifeng, the Chairman of
China’s apex planning body — the National Development and Reform Commission (NDRC) on Friday to
discuss the expansion of Chinese investments in Pakistan.

Head Office: 127, Zone II, MP Nagar, Bhopal |+91-7676564400| https://www.toprankers.com Page 10 of 36
Pakistan’s State Minister and Chairman of Board of Investment Muhammad Azfar Ahsan and He signed the
Framework Agreement on Industrial Cooperation which aims to attract Foreign Direct Investment (FDI),
promote industrialisation and development of economic zones, and initiate, plan, execute and monitor projects,
both in public as well as private sector, Pakistan’s state-run Associated Press of Pakistan news agency reported.
https://indianexpress.com/article/world/pakistan-signs-new-pact-with-china-to-kick-off-second-phase-of-cpec-
7757306/

36. In which of the following years did the China-Pakistan Economic Corridor launched?
(a) 2012 (b) 2015 (c) 2017 (d) 2019

37. China has proposed construction of which of the following as an extension of CPEC in
Afghanistan?
(a) Khyber Pass motorway (b) Khunjerab Pass
(c) Peshawar-Herat Motorway (d) Peshawar-Kabul motorway

38. Which of the following is not one of the objections raised by India against CPEC?
(a) China’s insistence on establishing the CPEC project through PoK is seen by India as infringing its
sovereignty.
(b) CPEC can decrease tension between Pakistan and India on Kashmir dispute.
(c) China’s increasing footprints in the South Asian region is detrimental to India’s strategic hold.
(d) China is building roads and infrastructure in the disputed territory of Gilgit-Balistan, which is under
Pakistan’s control but which India claims as a part of Jammu and Kashmir.

39. Which of the following is TRUE about the Belt and Road Initiative?
(a) President Xi Jinping launched BRI in 2015.
(b) The initiative was incorporated into the Constitution of China in 2017.
(c) The Chinese government calls the initiative “a bid to enhance regional connectivity and embrace a brighter
future.”
(d) The project has a target completion date of 2049, which will coincide with the centennial anniversary of the
People’s Republic of China (PRC)’s founding.

40. In which year did Pakistan bought Gwadar Port from Oman?
(a) 1951 (b) 1958 (c) 1961 (d) 1966

Passage (Q.41-Q.45): Two new Ramsar sites — Khijadiya wildlife sanctuary in Gujarat and [1] in Uttar Pradesh
— were announced on the occasion of World Wetlands Day at Sultanpur national park in Gurgaon Wednesday.
Forest and Wildlife Department officials said India has a network of [2] such sites, the highest in South Asia,
covering 10,93,636 hectares.

The Union Environment Ministry had notified the Sultanpur National Park and Bhindawas wildlife sanctuary in
Jhajjar as Ramsar sites — the first two such sites in Haryana — in August 2021. Ramsar sites are wetlands of
international importance as per UNESCO’s 1971 Convention on Wetlands held in Ramsar, Iran. Officials said
the certification brings visibility to ecologically sensitive wetlands and helps in conservation.
https://indianexpress.com/article/cities/delhi/two-more-ramsar-sites-announced-on-world-wetlands-day-
7753955/

41. Which of the following will replace [1] in the above passage?
(a) Haiderpur Wetland
(b) Bakhira wildlife sanctuary
(c) Parvati Aranga Wildlife Sanctuary
(d) Nawabganj Bird Sanctuary
Head Office: 127, Zone II, MP Nagar, Bhopal |+91-7676564400| https://www.toprankers.com Page 11 of 36
42. Which of the following is TRUE about Wetland?
(a) Wetlands are ecosystems saturated with water, either seasonally or permanently.
(b) They cover around 6% of the Earth’s land surface.
(c) 40% of all plant and animal species live or breed in wetlands.
(d) All of the above

43. Which of the following will replace [2] in the above passage?
(a) 46 (b) 47 (c) 48 (d) 49

44. Which of the following is NOT TRUE about Ramsar Sites?


(a) The Convention on Wetlands (Ramsar Convention) is the intergovernmental treaty that provides the
framework for the conservation and wise use of wetlands and their resources.
(b) It came into force in 1975
(c) India enforced the Ramsar convention in 1975
(d) In 1981, Chilika Lake was designated the first Indian wetland of international importance under the Ramsar
Convention.

45. Which is the largest Ramsar Site in India?


(a) Renuka Wetlands in Himachal Pradesh
(b) Chilika Lake in Odisha
(c) Sundarban Wetland
(d) Asan Conservation Reserve in Uttarakhand

Passage (Q.46-Q.50): The government does not subscribe to [1] views, its rankings and disagrees with the
conclusions it has drawn by ranking India [2] on its World Press Freedom Index, minister of state for home
affairs Nityanand Rai told Parliament on Tuesday.

In response to Congress member Manish Tewari’s query in the Lok Sabha about the latest ranking, Rai said
India’s disapproval is based on “very low sample size, little or no weightage to fundamentals of democracy,
adoption of a methodology which is questionable and non-transparent, lack of clear definition of press freedom
etc.”

With reference to Tewari’s question on the closure of Kashmir Press Club on January 18, Rai said there is no
registered body by that name. He added there is no elected managing body of the club since it, as a registered
body, has ceased to exist and failed to register itself under the Societies Registration Act, 1860.
https://www.hindustantimes.com/india-news/world-press-freedom-index-govt-rejects-india-s-ranking-
questions-methodology-101644382461067.html

46. Which of the following will replace [1] in the above passage?
(a) Reporters Without Borders
(b) Amnesty International
(c) Oxfam International
(d) Reports for Investigative journalism

47. The India Press Freedom Report, 2021 was recently released by who among the following recently?
(a) Report and Risk Analysis Group
(b) Rights and Risk Analysis Group
(c) Risk Assessment and Analysis Group
(d) Reporters Rights And Analysis Group

Head Office: 127, Zone II, MP Nagar, Bhopal |+91-7676564400| https://www.toprankers.com Page 12 of 36
48. Which of the following States tops the list of journalists/media organisations targeted as per the 2021 Indian
Press Freedom Report?
(a) Uttar Pradesh
(b) Jammu and Kashmir
(c) Madhya Pradesh
(d) Bihar

49. Which of the following will replace [2] in the above passage?
(a) 139 (b) 140 (c) 142 (d) 145

50. Freedom of press is a part and parcel of which of the following fundamental rights?
(a) Article 14 (b) Article 19(1)(a)
(c) Article 21A (d) Article 25(1)(b)

Passage (Q.51-Q.55): The Supreme Court on Thursday permitted the Central government to implement [1]
economically weaker sections (EWS) reservation in All India Quota seats in NEET undergraduate and
postgraduate seats for the academic year 2021-2022, pending a detailed hearing.

A bench of Justices D.Y. Chandrachud and A.S. Bopanna said: "We deem it necessary to allow the counselling
session to begin with the existing criteria for the identification of the EWS category. Judicial propriety would
not permit us to pass an interim order staying the criteria for determination of the EWS category."

It, however, said that meaning of EWS - the identification of the poor or the poorest, would have to be examined
through detailed hearing.
https://www.newindianexpress.com/nation/2022/jan/21/has-societal-implications-on-equality-supreme-court-
to-examine-ews-quota-criteria-in-march-2409532.html

51. Which of the following is NOT one of the criteria for reservation of EWS category?
(a) Family has gross annual income below Rs 8 lakh rupees are to be identified as EWSs for benefit of
reservation.
(b) 15 acres of agricultural land and above;
(c) Residential area of 1000 sq ft. and above;
(d) Residential plot of 100 sq. yards and above in notified municipalities;

52. Which of the following will replace [1] in the above passage?
(a) 08 per cent (b) 10 per cent (c) 12 per cent (d) 15 per cent

53. The amendment to added the EWS category to the Constitution of India added which of the following
articles to the Constitution of India?
(a) Article 15(5) (b) Article 15(6) (c) Article 16(6) (d) Both b and c

54. The Mandal Commission to "identify the socially or educationally backward classes" of India was formed in
which of the following years?
(a) 1975 (b) 1978 (c) 1979 (d) 1992

55. Which of the following cases in India proposed the threshold of 50% to the reservation in India?
(a) Indra Sawhney & Others vs Union of India
(b) Minerva Mills vs. the union of India
(c) Keshvanand Bharti vs. the State of Kerala
(d) Maneka Gandhi vs. Union of India

Head Office: 127, Zone II, MP Nagar, Bhopal |+91-7676564400| https://www.toprankers.com Page 13 of 36
Passage (Q.56-Q.60): The government has notified new rules under which the upper limit for termination of a
pregnancy has been increased from [1] to 24 weeks for certain categories of women.

According to the Medical Termination of Pregnancy (Amendment) Rules, 2021, these categories include
survivors of sexual assault or rape or incest, minors and women whose marital status changes during an ongoing
pregnancy (widowhood and divorce) and women with physical disabilities.

The new rules also cover mentally ill women, cases of foetal malformation that has a substantial risk of being
incompatible with life or if the child is born it may suffer from such physical or mental abnormalities to be
seriously handicapped and women with pregnancy in humanitarian settings or disaster or emergency situations
as may be declared by the government.
https://www.thehindu.com/news/national/new-rules-allow-abortion-till-24-weeks-of-pregnancy-for-certain-
categories-of-women/article36982316.ece

56. In which of the following cases, abortion is allowed even after the prescribe limit in the act?
(a) Women with physical disability
(b) Marital status change during pregnancy
(c) Foetal malformation
(d) Rape

57. Which of the following is NOT TRUE about the MTP Rules?
(a) Only doctors with specialisation in gynaecology/obstetrics can perform abortions.
(b) According to the Bill, the “name and other particulars of a woman whose pregnancy has been terminated
shall not be revealed”, except to a person authorised by law.
(c) The Bill permits abortion to be allowed up to 20 weeks on the opinion of just one medical practitioner.
(d) To terminate pregnancies between after 24 weeks, the opinion of two doctors are required.

58. Which of the following will replace [1] in the above passage?
(a) 16 (b) 18 (c) 20 (d) 22

59. Which of the following is false about MTP Act, 1971?


(a) The Medical Termination of Pregnancy Act, 1971 allowed for aborting the pregnancy by medical doctors
(with specified specialisation) on certain grounds.
(b) Termination is allowed at any point during the pregnancy if there is an immediate necessity to save the
woman’s life.
(c) Termination is permitted only when continuance of the pregnancy would involve a risk to the life of the
pregnant woman, cause grave injury to her mental or physical health or in the case of foetal abnormalities.
(d) Unmarried women are covered under the act.

60. IN which of the following landmark judgement of United States Supreme Court allowed abortion?
(a) Marbury v. Madison
(b) McCulloch v. Maryland
(c) Gibbons v. Ogden
(d) Roe v. Wade

Head Office: 127, Zone II, MP Nagar, Bhopal |+91-7676564400| https://www.toprankers.com Page 14 of 36
Passage (Q.61-Q.65): The One Sun One World One Grid Declaration (OSOWOG) was jointly released by Prime
Minister Narendra Modi and UK Prime Minister Boris Johnson at the COP26 Climate Meet in [1].

Realising the vision of OSOWOG through interconnected green grids can be transformational, enabling all
nations to meet the targets of the Paris Agreement to prevent dangerous climate change, the declaration said.
These efforts can stimulate green investments and create millions of good jobs. By sharing the sun's energy, one
can help to build a more peaceful and prosperous world, the declaration added.

One of the big challenges for implementation will be maintaining a stable grid over a large geographical area.
Electricity Grid is vulnerable to accidents, weather, and cyber-attacks that are prone to increase and disrupt the
electricity supply on mass scale. The mechanism of cost-sharing will also be challenging as participating
countries are both rich and poor nations.
https://www.thehindubusinessline.com/blexplainer/bl-explainer-all-about-the-one-sun-one-world-one-grid-
initiative/article37486937.ece#:~:text=OSOWOG%20is%20the%20solution%20to,where%20it%20is%20day
%20time).

61. Which of the following will replace [1] in the above passage?
(a) Belgium (b) Geneva (c) Rome (d) Glasgow

62. Which of the following is one of the objectives of the One Sun One World One Grid Declaration?
(a) Its objective is to aid in developing a worldwide grid through which clean energy can be transmitted
anywhere, anytime.
(b) It also aims to help in reducing storage needs and enhancing the viability of solar projects.
(c) Its ultimate goal is to reduce carbon footprints and energy costs.
(d) All of the above

63. The idea of One Sun One World One Grid initiative was first floated at which of the following platforms?
(a) COP 21
(b) First Assembly of International Solar Alliance
(c) Third Assembly of International Solar Alliance
(d) G20 Summit 2019

64. The OSOWOG initiative is also known as which of the following in the UK?
(a) Green Grids (b) Solar Grids (c) Clean Grid (d) 24*7 Grid

65. Which of the following is NOT TRUE about International Solar Alliance?
(a) The ISA is an inter-governmental organisation formed by India and France.
(b) It comprising 101 members, to accelerate the global adoption of solar power.
(c) The ISA and the World Bank are also helping in executing the OSOWOG initiative.
(d) The United Nations General Assembly has rejected the request of International Solar Alliance to join them
as Observer member.

Head Office: 127, Zone II, MP Nagar, Bhopal |+91-7676564400| https://www.toprankers.com Page 15 of 36
SECTION – C: LEGAL REASONING

Directions (Q.66 – Q.105): Read the comprehensions carefully and answer the questions based on it.
Passage (Q.66-Q.71): High Court stated that it is trite law that even if the weapon of offence is shown after
snatching had taken place for running away along with snatched article, an offence under Section 397 IPC is
attracted and if the offender fails to acquire the property, it will be an attempt to robbery.
Section 390 CrPC provides that in a robbery, there is either theft or extortion. It is further provided that theft is
‘robbery’ if, in order to committing of the theft or in committing the theft, or in carrying away or attempting to
carry away property obtained by theft, the offender, for that end, voluntarily causes or attempts to cause to any
person death or hurt or wrongful restraint, or fear of instant death or of instant hurt or of instant wrongful restraint.
When the offender uses deadly weapon while committing robbery or dacoity, he is punished under Section 397.
Extortion is “robbery” if the offender is in the presence of the person put in fear, and commits the extortion by
putting that person in fear of instant death, of instant hurt, or of instant wrongful restraint to that person or to
some other person, and, by so putting in fear, induces the person, so put in fear then and there to deliver up the
thing extorted.
The Bench stated that it is not essential that weapon of offence should be recovered to prove the nature of the
weapon used and that a deadly weapon was used at the time of the commission of the offence, however, the
prosecution was required to prove the nature of the weapon of offence used especially in the case of knife or
blade.
Court held that since from the evidence of the prosecution witnesses the size and sharpness of the blade was not
proved, the prosecution failed to prove that the appellant used a deadly weapon.

(Extracted with requisite revisions and edits from ‘Whether a ‘blade’ would be covered under S. 397 IPC as a
deadly weapon? Del HC explains in view of settled position of law’by Devika Sharmaat
https://www.scconline.com/blog/post/2022/02/04/whether-a-blade-would-be-covered-under-s-397-ipc-as-a-
deadly-weapon/)

66. In reference to above stated passage, which is true about deadly weapon:
(a) Knife is a deadly weapon.
(b) Deadly weapon must be used for commission of Robbery.
(c) Weapon which is deadly must be produced before court.
(d) Some weapons become deadly depending on its nature.

67. Mahesh went to Rakesh and asked him for money so that he can pay the expenses for his mother’s medical
treatment. Rakesh denied giving him the amount and asked to leave the place. Mahesh later on calls Rakesh and
says that he will injure him badly if he does not give him the money. He says that his friend is present in his
vicinity and has a knife with which he will hurt him. Decide whether Mahesh is guilty of his act:
(a) Mahesh is guilty for Robbery under Section 392.
(b) Mahesh is not guilty for Robbery.
(c) Mahesh is guilty under Section 397.
(d) Mahesh did not injure Rakesh and did not commit any offence.

68. Mohan entered into the house of Raka. He saw that nobody is the home and stole jewelry from his house. While
he was running away with the jewelry Raka saw him. He tried to stop Mohan but failed to do so. To stop Raka,
Mohan kept the jewelry aside on the road and picked up a rock and threw it at Raka. The stone did not hit Raka.
Decide whether Mohan is guilty:
(a) It is not robbery as Mohan kept the stolen jewelry aside.
(b) Mohan just committed theft as he stole the Jewelry from Raka.
(c) Mohan committed robbery as he was running away with stolen jewelry.
(d) It is punishable under Section 397 as Mohan used stone which is a deadly weapon.

Head Office: 127, Zone II, MP Nagar, Bhopal |+91-7676564400| https://www.toprankers.com Page 16 of 36
69. A sees that B is going to market is carrying money with him. A wanted to get the money from B and pushed B
down asking him for money. He says that B should deliver the money to him and then only he will allow him to
go. While A was holding him down him down, B sees the knife that A was carrying in his pocket. B somehow
loosens the grip of B and runs away. B later on files a complaint against A. Decide for which act can A be
punished:
(a) A is guilty of attempt to robbery.
(b) A is guilty of robbery.
(c) A is guilty of robbery with deadly weapon.
(d) A is guilty of extortion.

70. X approaches Y with a gun in his pocket and tells Y that he does not give all his valuables to X, he will shoot
him. He tells that Y he has a gun with him. Y was scared and therefore, unwillingly handed over his property. X
went away after taking all the property from Y. Y saw Z around and asked for his help to catch X who was
running away. When Z runs towards X, he took out his gun and pointed it towards Z and threatened to shoot
him. Decide the offence for which X is guilty:
(a) He is guilty of Robbery with deadly weapon as he used the gun.
(b) He is guilty of Robbery done in furtherance of extortion.
(c) He is guilty of robbery done in furtherance of theft.
(d) He is not guilty of robbery as he did not use the gun to threaten Y to give up his valuables.

71. In furtherance of above passage, decide which is false in relation of robbery:


(a) Robbery can be committed even without fear of instant death, instant hurt or instant wrongful restraint.
(b) Theft or extortion is a pre-requisite for commission of robbery.
(c) A person must be put in fear while committing robbery by extortion.
(d) While committing Robbery in Section 397 weapon should be used before or at the time of stealing the
property.

Passage (Q.72-Q.77): “Every agreement by which any one is restrained from exercising a lawful profession,
trade or business of any kind is to that extend void. There is one exception to this rule-that if the goodwill of a
business has been sold, an agreement to refrain from carrying on similar business, if it appears to the Court to be
reasonable, would be protected and would be enforced.”
In Supreme Court’s decision of Superintendence Company of India v. Krishna Murgai, the Supreme Court while
discussing the objective behind Section 27 of the Act analyzed the difference of negative covenant between an
employer-employee and a seller-purchaser and stated that a negative covenant between the employer – employee,
pertains to performance of personal service which is altogether different in substance from purchase and will
have vastly different social and economic implications. The essential line of distinction is that the purchaser is
entitled to protect himself against competition on the part of his vendor, while the employer is not entitled to
protection against mere competition on the part of his servant. A restrictive covenant ancillary to a contract of
employment is likely to affect the employee’s means or procuring a livelihood for himself and his family to a
greater degree than that of a seller, who usually receive ample consideration for the sale of the goodwill of his
business.
An employment contract generally includes restraint of trade clause to protect the interest of the employer after
an employee leaves their organization or business for the following reasons:
 Non-compete clause
 Non-solicitation clause
 Non-recruitment clause
 Confidentiality clause
In Independent News Service (P) Ltd. v. SucheritaKukreti, in the context of Section 27 of the Contract Act, it
was held that the right saved thereby to be a facet of Article 21 of the Constitution of India.

Head Office: 127, Zone II, MP Nagar, Bhopal |+91-7676564400| https://www.toprankers.com Page 17 of 36
Extracted with requisite revisions and edits from ‘For a contract to be enforceable, restraint of trade clause
must be reasonable: Post-termination non-compete clauses are permissible in employment contracts under S.
27 of Contract Act? District Court explains’ by Devika Sharma
athttps://www.scconline.com/blog/post/2022/02/03/for-a-contract-to-be-enforceable-restraint-of-trade-clause-
must-be-reasonable/)

72. M sold unlicensed guns for his livelihood. He used to check the details, documents and took other necessary
precaution before selling the gun to a person. His mother was worried that such a business might threaten the
safety of M. M explained to his mother that he sells the gun after doing a background check. His mother did not
listen to him and asked him to sign a contract with her promising that he will stop selling gun. M unwilling
signed the contract due to the pressure from his mother. Decide whether the contract is valid:
(a) No, as the contract restrict M from selling gun and therefore, is void under Section 27.
(b) No, as no contract was formed as M did not give free consent and signed contract due to the pressure from
her mother and therefore, is void.
(c) Yes, as M’s mother just stopped his from selling gun and did not stop him from doing any other trade.
(d) Yes, as the trade of M is not lawful.

73. Mehul is an advocate and he was hired by Tarun as his advocate. Tarun was very concerned about his privacy
and did not want his information to be leaked. He asks Mehul to sign a contract to act as his advocate and to not
work as the advocate of any person in future. Decide the validity of the contract:
(a) The contract is void.
(b) The contract is valid as Tarun is just trying to protect his interest.
(c) The contract is void to the extent on restricting Mehul.
(d) The contract can be declared void by Mehul.

74. In accordance with above stated passage, decide which statement is true:
(a) Trade, profession and business cannot be restricted at all.
(b) The restriction imposed on trade must be reasonable.
(c) Imposition of restriction does not violate any right.
(d) The restriction imposed must be reasonable to the parties to contract.

75. John was hired by a multi-national company after his graduation. He signed a contract with the company without
going through the contract completely. One of the terms of the contract was that John cannot ever work in the
same line of business as his company after he leaves the job. When John learned about the term of the contract,
he was furious. He contended that the contract is void as the condition restricted his profession. Decide the
validity of the contract:
(a) John has already signed the contract and will be valid.
(b) The contract is void as it restricted the profession of John.
(c) The contract is valid as the company as the employer can impose such restriction.
(d) The contract is void as the restriction imposed by company is unreasonable.

76. Which of the following is true about restriction imposed by employer on employee:
(a) Employer cannot restrict his present employees.
(b) Restriction must be reasonale.
(c) Employer cannot restrict regarding confidential information
(d) Employer can impose restriction by a contract after the person has stopped being the employee.

Head Office: 127, Zone II, MP Nagar, Bhopal |+91-7676564400| https://www.toprankers.com Page 18 of 36
77. Decide which is true about imposition of restriction by employer on employee and seller on purchaser:
(a) The degree of affect on the interest of parties can be ground to distinguish employer-employee contract from
seller-purchaser contract.
(b) Competition can be involved as a sole ground of restriction by the employer.
(c) The restriction by seller creates a positive right in favor of purchaser.
(d) There exists no social and economical implication in restriction imposed by purchaser on seller as he is
getting consideration for the goodwill of his business.

Passage (Q.78-Q.83): The Supreme Court's judgment in Gargi v. State of Haryana which had elaborated on
circumstantial evidence, which is not the first choice of evidence, stating that "circumstantial evidence is the one
whereby other facts are proved from which the existence of the fact in issue may either be logically inferred or
at least rendered more probable."
Furthering referring to the Apex Court's judgment in Sharad Birdhichand Sarda v. State of Maharashtra where
the panchsheel principles pertaining to acceptance of circumstantial evidence were laid down, the bench
reiterated those
 The circumstances from which the conclusion of guilt is to be drawn should be fully established
 The facts so established should be consistent only with the hypothesis of the accused's guilt (and no other)
 Circumstances should be of conclusive nature and tendency
 Such circumstances should exclude every other hypothesis
 The chain of evidence must be so complete that it leaves no reasonable ground to conclude or even point to
the innocence of the accused and must show in all human probability that the offence has been committed by the
accused.
Further holding that "extra-judicial confession cannot be the sole basis of conviction and cannot be relied on
when surrounding circumstances are improbable and create suspicion", the bench noted that since the chain of
events does not lead to the result of accused committing the crime of murder, the trial court had erred in
convicting the accused and the conviction of the accused was set aside and miscellaneous applications pending
stood closed.
(Extracted with requisite revisions and edits from ‘Judges To Tread Cautiously In Circumstantial Evidence,
Can't Allow Conjectures & Suspicion To Take Place Of Proof: Telangana High Court' by Rashmi Bagriat
https://www.livelaw.in/news-updates/telangana-high-court-circumstantial-evidence-proof-judges-to-tread-
cautiously-191742)

78. In furtherance of above passage, decide the correct statement relating to circumstantial evidence:
(a) Circumstantial evidence is relied upon when there is no direct evidence.
(b) Circumstantial evidence is needed in every case irrespective of the availability of direct evidence.
(c) A person cannot be convicted solely on the basis of circumstantial evidence.
(d) The court cannot deny accepting circumstantial evidence.

79. The case before the court is that whether Mahesh killed Rakesh or not. Rakesh’s body was found after he was
shot. There exists no eye witness to prove the same. The police find that Mahesh was present at the same place
where Rakesh’s dead body was found. It was also found that Mahesh had made an attempt in the past to injure
Rakesh and wanted to take revenge from Rakesh. The police also found that Mahesh called Rakesh before his
death and also went to market to purchase a knife. Decide whether there exists circumstantial evidence to convict
Mahesh:
(a) Yes, as the circumstances establish a chain of evidence which prove that Mahesh is guilty.
(b) Yes, as the Mahesh being present at the same place fully establish guilt conclusively.
(c) No, as there exist discrepancy in the chain of evidence and therefore, it creates a doubt.
(d) Yes, as Mahesh made attempt in past to injure Rakesh which proves his intention for murder.

Head Office: 127, Zone II, MP Nagar, Bhopal |+91-7676564400| https://www.toprankers.com Page 19 of 36
80. In accordance with the above passage decide which statement is true:
(a) Circumstantial evidence creates no connection in between the fact in issue and the facts proved by it.
(b) For court to rely on circumstantial evidence just proving any of the facts is enough.
(c) Circumstantial evidence establishes a direct relation between proving a fact and existence of fact in issue.
(d) Circumstantial evidence establishes an inverse relation between proving a fact and existence of fact in issue.

81. The police is investigating the theft at X’s house. The police find that Z and Y were aware of the fact that X is
not at his place. Z has a criminal record of theft in past. The police are sure that the theft was committed by a
single person. Z was aware of the fact that where X has stored his valuables at his house. Y was nearby X’s place
at the time of theft. However, there is no evidence to show that Y entered X’s house. Decide who can be guilty
for theft:
(a) Z as he has past criminal record and Z was aware of X’s house.
(b) Z and Y both can be the accused due to which none of them can be deemed guilty.
(c) Y as guilty as he was aware of X’s house and was present nearby at the time of theft.
(d) Z and Y both are accused and therefore, both are guilty.

82. Arun is suspected to have killed Hemant. The facts show that Arun is a person of good character and have always
regarded Hemant as his brother. Arun was found to be having bought knife from a store nearby. When Arun was
returning back to his house his neighbor found that his clothes were covered in blood and Arun was very anxious.
Arun later on burnt his clothes late at night which was seen by his neighbor. Decide whether there exist evidence
to prove Arun’s guilt:
(a) No, as the circumstances show that Arun treated Hemant like his brother and therefore, his guilt cannot be
conclusively established.
(b) No, as the circumstances give rise to two hypothesis one of the guilt of Arun and another of innocence of
Arun.
(c) No, as there is absence of chain of evidence to prove guilt of Arun beyond reasonable doubt.
(d) Yes, as the circumstances prove beyond reasonable doubt the guilt of Arun.

83. In furtherance of above stated passage decide the false statement regarding circumstances:
(a) The circumstances can relate to anything to prove any possibility.
(b) Circumstances should show the absence of all other possibilities.
(c) Circumstances relevant to fact in issue are to be considered for establishing guilt.
(d) Circumstances must be consistent with the confession so that it can be relied upon.

Passage (Q.84-Q.89): Commission noted that as per the Supreme Court decision in DrMukhtiar Chand v. State
of Punjab, Ayurveda, Siddha, Unani and Homeopathy practitioners can prescribe allopathic medicines only in
those states where they are authorized to do so by a general or specific order made by the State Government
concerned.
Further, the Coram expressed that, Few State Governments have authorized AYUSH doctor(s) it by some special
order(s) to prescribe medicines of allopath, but in our view, that does not authorize the doctor to deviate from
the standard of care which results into wrong diagnosis and prescribe wrong medicines.
Coram relied on the Supreme Court’s decision in PoonamVerma v. Ashwin Patel, wherein it was held that “the
doctor must not only be qualified, but he must also be registered with the appropriate Medical Council in order
to practice as a doctor. A homoeopath would not have knowledge about allopathic medicines and its drug actions,
so administration of allopathic treatment by a homoeopath would be proof enough to establish negligence”.
Coram noted the 4 points on which the OP 2 failed:
 Failure to correctly diagnose the condition as SJ syndrome but treated the patient for Measles.
 The dose of steroid 4mg Dexa was inadequate for the patient; the dosage should be calculated as per kg of
body weight.
 For treatment of SJ syndrome the appropriate drugs of choice would be Cyclosporine with
Methylprednisolone and the dosage to be per kg body weight of the patient.
Head Office: 127, Zone II, MP Nagar, Bhopal |+91-7676564400| https://www.toprankers.com Page 20 of 36
 There was a delay and wrong referral of the patient to the physician instead of a skin specialist.
Therefore, the patient deserved enhanced just and reasonable compensation.

(Extracted with requisite revisions and edits from ‘Minor treated for “Measles” instead of “Stevens-Johnson
Syndrome” due to wrong diagnosis and leading to medical negligence: Read detailed report on NCDRC’s
decision’ by Devika Sharmaathttps://www.scconline.com/blog/post/2022/01/28/medical-negligence-6/)
84. Alok is a 27 years old man. He has certain medical condition due to which he is unable to perform his daily
activities. He consults Mahesh who is an expert in ayurvedic medicine. Mahesh prescribed him certain
ayuervedic medicines. He further said that if Alok does not get relief from the medicine, he should take some
antibiotic to relieve his pain. Mahesh suggested the names of the antibiotics to Alok orally and told him that he
should once talk to his doctor once before talking the medicine. Decide whether act of Mahesh is correct:
(a) No, as he acted negligently.
(b) Yes, as Mahesh just wanted to relive the pain of Alok and acted in good faith.
(c) No, as he should not prescribe antibiotic to Alok.
(d) Yes, as he just made a suggestion did not prescribe the antibiotic to Alok.

85. In the last question, if Mahesh wrote on a slip prescribing certain ayuervedic medicine and an antibiotic. Mahesh
prescribed it in good faith without any intent to cause him harm and to the best of his knowledge. Decide whether
the act of Mahesh is correct in accordance to the general tone of the passage considering his State has not allowed
the same:
(a) Yes, as Mahesh acted in good faith.
(b) No, as Mahesh is not authorized to prescribe antibiotic.
(c) Yes, Mahesh acted as per his knowledge and to benefit Alok.
(d) No, as Mahesh should have gotten consent of Alok.

86. In accordance with the above passage, decide which statement is correct in regard to medical negligence:
(a) It can be only punished by imprisonment.
(b) Medical negligence will not occur when state government authorizes anayuerveda practitioner.
(c) Medical negligence may occur even after authorization from state government to the practitioner.
(d) It won’t occur when practitioner has knowledge.

87. A is a very bright student and wanted to become a doctor. He got admitted into the medical school. One day
while he was on his way to college he saw that a person fainted and to make the person recover he administered
a medicine to him. Later on the patient suffered from side effects due to the medicine and he sued A for medical
negligence. Decide whether the patient would succeed:
(a) Yes, as A is not a registered doctor.
(b) No, as A administered him medicine to save him.
(c) No, as A did not prescribe medicine to the patient.
(d) No, as A acted best to his ability.

88. Mukund is a homeopathy practitioner in state of V. The state published a general order allowing homeopathy
practitioner in the state to prescribe allopathic medicine to their patients. Mukund never studied about allopathic
medicine and did not have knowledge about it. He prescribed allopathic medicine to one of his patients. The
patient suffered after consuming the medicine and sued Mukund for medical negligence. Decide whether the
patient would succeed:
(a) Yes, as Mukund if a homeopathy practitioner.
(b) Yes, as Mukund had no knowledge about allopathic medicine.
(c) No, as state government authorized Mukund to prescribe allopathic medicine.
(d) No, as the patient consumed the medicine with his choice.

Head Office: 127, Zone II, MP Nagar, Bhopal |+91-7676564400| https://www.toprankers.com Page 21 of 36
89. In the last question, if Mukund is of 29 years old and government order authorized the practitioner above 30
years. Mukund was aggrieved by the order of government and challenged it. Whether government make such an
order:
(a) No, as government cannot just allow limited people.
(b) Yes, as government can do anything they wish.
(c) No, as government cannot authorize a homeopathy practitioner to do something they have not studied.
(d) Yes, as the government can make specific order.

Passage (Q.90-Q.95): Section 52 incorporates the doctrine of lispendens and stipulates that during pendency of
any suit or proceeding in which any right to immoveable property is, directly or specifically, in question, the
property, which is the subject matter of such suit or proceeding cannot be transferred or otherwise dealt with, so
as to affect the rights of any other party to such a suit or proceeding. It is to maintain the status quo unaffected
by the act of any party to the litigation pending its determination.
Order 39 Rules 1 and 2 deals with cases in which temporary injunction may be granted. It is a discretionary relief
exercised by the courts.
The distinction between the two is that an order of temporary injunction is pre-emptive in nature restraining the
act of alienation by party to the suit where there is such a danger, whereas Section 52 T.P. Act comes into play
after the alienation takes place during pendency of the suit. If an order of temporary injunction is passed and
transfer is restrained, the question of applicability of Section 52 of T.P. Act will not arise as then there will be
no transfer pending litigation.
In view of the primary object of grant of temporary injunction to maintain the status quo till the adjudication of
the rights of the litigating parties, the Court held that Section 52 of TP Act does not operate as a bar to grant of
temporary injunction under Order 39 Rules 1 & 2 CPC, in the discretion of trial court, on fulfillment of pre-
conditions (prima facie, balance of convenience and causing irreparable loss or injury in his favour) for grant of
temporary injunction, restraining alienations as well.

(Extracted with requisite revisions and edits from ‘Section 52 Transfer Of Property Act Does Not Bar Temporary
Injunction Against Alienation Of Property: Andhra Pradesh High Court’ by JagritiSanghi at
https://www.livelaw.in/news-updates/andhra-pradesh-high-court-section-52-transfer-of-property-act-bar-to-
grant-of-temporary-injunction-190679)

90. Mehul and Manisha were indulged in a suit for partition of their father’s property. The general practice in their
family is that the daughter does not acquire the property of the father. Mehul wanted some money so that he
could pay for certain expenses of his deceased father. Mehul thought of selling the house which has been
succeeded from their father. Mehul sold the house and Manisha challenged the act of Mehul. Decide whether
Manisha would succeed:
(a) Yes, as during pendency of suit Mehul cannot sell the house.
(b) No, as since Mehul sold the house to pay expenses of his deceased father.
(c) No, as Mehul had right over the house.
(d) No, as in accordance with general practice Manisha cannot acquire property of her father.

91. Nitin and Yash have a suit going on between them regarding a land and garden. The court is hearing the suit on
daily basis and is trying to resolve the dispute as soon as possible. Yash had right over the garden concerned.
Mahi learnt that Nitin who has the possession of the property was disposing the garden. Mahi was not having
any right on the garden but filed a case challenging the transfer. Decide whether Mahi would succeed:
(a) Yes, as Mahi’s right has been affected.
(b) No, as Mahi can file a suit later on.
(c) Yes, as transfer was made during pendency of suit.
(d) No, as Mahi is not a party to the suit.

Head Office: 127, Zone II, MP Nagar, Bhopal |+91-7676564400| https://www.toprankers.com Page 22 of 36
92. In reference to the above passage, decide which of the following the correct relationship between Section 52 and
Order 39:
(a) They both are contradictory to each other.
(b) They are complementary to each other.
(c) They both cannot be claimed as a relief is a same suit.
(d) They both can be claimed as a preventive measure.

93. Anand and Renu are a married couple. They planned to get a divorce and filed a suit for divorce. At the same
time Renu filed a suit claiming the ownership of car which was in the possession of Anand. Anand did not want
Renu to get the car and therefore, intentionally sold the car to another person. Renu filed a case challenging the
sale under Section 52. Decide whether she would succeed:
(a) No, as Renu should wait for the case of divorce to be decided before filing the case for ownership of car.
(b) Yes, as transfer was made during pendency of suit.
(c) Yes, as transfer was made to intentionally cause injury to Renu’s right.
(d) No, as the property involved in the pending suit is car.

94. A and B has a suit going in between them regarding a land title. A contended that he rented the land to B. When
he demanded the land back from B, he was denied due to which he filed a case. A also applied to court to grant
temporary injunction against B restraining him from entering on the land. A presented the document in relation
to the land to depict his ownership. A contended that if he is not granted injunction, he will lose his source of
income. He also notified court that B has acquired the income arising from the land. Decide whether Court can
grant temporary injunction:
(a) No, as there is no prima facie case.
(b) Yes, as all the requisites has been fulfilled.
(c) No, as there in no balance of convenience in favour of A.
(d) No, as no injury is caused to A.

95. In accordance with above passage which statement is correct regarding temporary injunction:
(a) It is a matter of right which Court must grant.
(b) Temporary injunction makes changes in the rights of parties.
(c) It is granted after transfer has taken place.
(d) Court must be satisfied that all the pre-requisites of temporary injunction have been fulfilled before granting
it.

Passage (Q.96-Q.100): Ms. Tanu Bedi, Counsel for appellant had vehemently argued that the defense counsel
was not given any opportunity to cross-examine the victim in the trial court and thus, the testimony of the victim
could not be relied on for convicting the accused-appellant. In reply the learned Counsel stated that they "had
slept over their right to do so" as they did not submit any questionnaire at the time of examination of the victim,
nor did they make an application for cross-examination. They did not even raise any objection and argument
regarding the same during the trial either.
After scrutinizing the submissions made, the Court had to look into question: Whether the testimony of a child
be the sole reason for the conviction of the accused-appellant? The Court then proceeded to peruse S.33 of
POCSO Act, noting the legislature's wisdom of providing a special, specific procedure for recording the evidence
of child witnesses and clarified that "in cases of sexual assault against Children, the first and most important
piece of evidence is always the statement of the child victim Themselves."
Referring to the apex court's judgments in Sanjay Kumar Valmiki v. State which held that if the trial court is
satisfied that the child witness before it is unlikely to be tutored and has deposed of their own will and volition,
then the evidence given by such child witness should be treated with the same regard as that of any other witness
as given in Sec 118, the High Court averred that there is no embargo on child testimony in S.118 of Evidence
Act, that talks about ‘who may testify’ and the duty is cast on the courts regarding allowing or disallowing such
evidence on the grounds that child does not understand the questions put to him or to provide rational answers
Head Office: 127, Zone II, MP Nagar, Bhopal |+91-7676564400| https://www.toprankers.com Page 23 of 36
to such questions and if the answers to these questions are in negative then there is no justification to disregard
such testimony. Thus, the court stated that the Child witness's testimony was substantial and enough under S.118
of the Evidence Act.
(Source : No Embargo In Treating Testimony Of A Child Witness As The Sole Basis For Conviction: Punjab
And Haryana High Court, livelaw)

96. Ramu is a ten-year-old boy who is unable to speak. Every morning, he goes with his mother to get water from
a nearby well. When they both went to the well one day, they encountered an impediment in the form of a large
tree that had fallen owing to severe rain. She told Ramu to stay put until she returns. Ramu eventually heard his
mother's screams from the other end, but due to the tree blocking his view, he couldn't see what was going on.
Though he heard his uncle Chetan's voice, he couldn't be sure because he couldn't see his face. Raju was
summoned by the court since he was the only witness present at the scene. Ramu heard everyone saying that
chetan might have killed his mother. Chetan killed Ramu's mother, according to Ramu's testimony. Is the court
likely to accept Ramu's statement under section 118 of the Evidence Act?
(a) The court will not allow Ramu’s testimony as he can’t speak.
(b) The court will not allow Ramu’s testimony as the statement was tutored due to his tender age.
(c) The court will allow the testimony of Ramu as he testifies in her testimony Chetan’s criminal liability.
(d) The court will allow Ramu’s testimony as he heard Chetan’s voice at the crime scene.

97. In the same circumstances as above, what procedure will the court employ to extract Ramu’s testimony?
(a) Direct and to-the-point questions are preferred.
(b) The question should be phrased in a way that the child witness can understand.
(c) The questions must be put forth in form of yes or no.
(d) When obtaining the evidence of a child witness, the court must ensure that the statement made by the child
is made of their own free will and volition.

98. Which of the following statements correctly depicts the essential of section 118 of the IEA?
(a) The testimony of child witness should always be supported by corroborative evidence.
(b) The testimony of a witness should be untutored and of free will.
(c) The testimony of child witness should be tutored and of free will.
(d) The testimony given by a major, sound mind is admissible in addition to Child witness's testimony is
substantial and enough under S.118 of the Evidence Act.

99. Which of the following is not true, based on your understanding on the subject of child witness as discussed in
the above passage:
I. In the case of rape or assault, a juvenile witness can also be subjected to cross examination by the defence
lawyer, and is not limited to judicial testimony.
II. In situations involving criminal offences against children, the first and most essential piece of evidence is
always the child victim's own statement.
III. The courts are tasked with allowing or disallowing such evidence on the basis that the child does not
understand or provide rational answers to the questions posed to him, and that if the responses to these
questions are negative, there is reason to reject such testimony.
IV. Upon court’s satisfaction, the testimony of a child witness can be considered substantial and sufficient under
S.118 of the Evidence Act.
(a) II & III
(b) 1&III
(c) II& IV
(d) ONLY III

Head Office: 127, Zone II, MP Nagar, Bhopal |+91-7676564400| https://www.toprankers.com Page 24 of 36
100. Chandini a 6th class student file a complaint against his sports teacher for sexual assault as he used to touch her
inappropriately on several occasion. She in her testimony stated “I know what is good touch and bad touch”.
Court was very impressed by the statement of Chandini as she had been raised well by her parents and has
knowledge of good and bad touch. Based on this testimony of Chandini the court convicted the teacher under
POCSO Act without taking any other substantial evidence on record. Can the statement provided by Chandini
stands acceptable under section 118 of IEA?
(a) Yes, it should be allowed because chandini's testimony is not tutored and given of her own free will.
(b) It is unacceptable because a court cannot convict an accused based solely on the testimony of a minor
witness.
(c) It is acceptable because the statement of the child victim has toalways be the first and most essential piece
of evidence in situations of sexual abuse against children and used to determine guilt.
(d) It is not acceptable because, because child testimony is not always reliable, the court should have reviewed
substantial evidence before convicting the accused.

Passage (Q.101-Q.105): The Delhi High Court has observed that an FIR filed under the Protection of Children
from Sexual Offences (POCSO) Act, 2012 cannot be quashed on the ground that the victim after attaining
majority decided to compromise the matter with the accused.
A single judge bench comprising of Justice Subramanian Prasad observed thus:

"Exercising jurisdiction under Section 482 Cr.P.C to quash an offence under POCSO Act would go against the
intention of the legislature which has brought out the special enactment to protect the interests of children. The
FIR cannot be quashed on the ground that the victim after attaining majority has decided to compromise the
matter with the accused."
The Court was dealing with a petition filed under sec. 482 CrPC seeking quashing of FIR registered under sec.
354 (Assault of criminal force to woman with intent to outrage her modesty), 354D (Stalking),
506 (Punishment for criminal intimidation), 509 (Word, gesture or act intended to insult modesty of a
woman), 34 (Common intention) of IPC and sec. 10 (Punishment for aggravated sexual assault) of the
POCSO Act. The petition was filed on the ground that both the prosecutrix and the petitioner have compromised
the matter.
The Court, relied on the Statements of Objects and Reasons of the POCSO Act which reads "heinous crime
like rape cannot be quashed by the High Court by exercising power under Section 482 Cr.P.C. even if the
prosecutrix and the accused have entered into a compromise."
In view of this, the Court dismissed the petition after observing that the Court was not inclined to quash FIR in
a case wherein petitioners were accused of sec. 10 of the POCSO Act.
(Source : POCSO- FIR Cannot Be Quashed On The Ground That Victim Decided To Compromise Matter After
Attaining Majority: Delhi High Court live law )

101. Rupesh gave his neighbour Mailini a ride because she was running late for work and had missed her bus. During
the ride, Rupesh made Malini very uncomfortable by repeatedly hitting the brakes in order to keep Malini from
holding him. Even after arriving at the office, she was disturbed by Rupesh's actions. In these circumstances,
can Malini file a complaint against Rupesh?
(a) Rupesh act will not be punished under section 10 of POCSO act for committing sexual assault on malini but
under IPC.
(b) Rupesh act will be punished under 354 of IPC for Assault of criminal force to woman with intent to outrage
her modesty.
(c) Rupesh act will not be punished under any penal provision as he had no intention to outrage her modesty.
(d) Rupesh act will be punishable under section 509 of IPC for act intended to insult modesty of a woman.

Head Office: 127, Zone II, MP Nagar, Bhopal |+91-7676564400| https://www.toprankers.com Page 25 of 36
102. A group of four people, A, B, C, and D, raped a girl named Jeenu aged 17 years 5 months as a form of retaliation
for her declining D's proposal. D did nothing but abetted A, B, and C for committing sexual assault on
Jeenu. Determine the culpability of all the alleged perpetrators:
(a) All the accused will be liable under penal provision of IPC for raping jeenu.
(b) D will be punished under POCSO as he is minor while others will be punished under provisions of IPC.
(c) All the accused will be punished under the POCSO act and provision of IPC will be applicable on them.
(d) Only A, B and C will be punished under POCSO act as D is not an accomplice in the crime only the abetter.

103. A petition was filed in the high court under section 482 for the quashing of a FIR filed under section 10 of the
POCSO Act (Punishment for Aggravated Sexual Assault). The prosecutrix said that the accused, a distant
relative, had begun to reside in her home and that, in the absence of her parents, the accused "started looking at
her with odd eyes." Furthermore, the accused, along with his other two nephews, allegedly threatened her with
death over her photographs and even threatened to kill her parents. The petition was filed after a settlement was
reached between the prosecutrix's mother and the defendants. Decide
(a) The court will quash the FIR as there was compromised made between prosecutrix mother and the
defendants.
(b) The court will not quash the FIR since a solution was reached between the prosecutor's mother and the
defendants rather than the prosecutor and the defendants.
(c) The court will quash the FIR as the victim is a minor and is represented by her mother.
(d) Even if the parties come to some agreement, the court will not dismiss the FIR.

104. Choose the correct statement:


(a) The POCSO act recognises the fact that any person including a child can be prosecuted as a sexual offender.
(b) Sections of the POCSO Act may not be added by the police in the First Information Report (FIR) whenever
a sexual offence is committed against a child.
(c) Court can quash an FIR under section 482 of CrPC.
(d) An FIR can charge someone with rape under section 376 of the IPC then it cannot charge under Pocso Act..

105. Assertion: Court can only quash an FIR if compromise is reached between the parties in case of offences
punishable under IPC.
Reason: If the parties have reached an agreement through free consent, an application under section 482 of the
CrPC can be submitted to dismiss the charges.
(a) Both A and R are false but R is not correct explanation of A .
(b) Both A and R are true and R is correct explanation of A.
(c) A is true but R is false.
(d) A is false but R is true.

Head Office: 127, Zone II, MP Nagar, Bhopal |+91-7676564400| https://www.toprankers.com Page 26 of 36
SECTION - D: LOGICAL REASONING

Passage (Q.106-Q.110): Fortunes of many political families are on the line in the upcoming five state assembly
elections and for a change these elections are seeing some family members on opposing sides of the political
divide. After a prolonged schism, Akhilesh Yadav has marginalised everyone else in his big family, which had
earned SP much public scorn for being a family enterprise. With no future in SP, his sister-in-law Aparna Yadav
has joined BJP. The Badal family domination of the Akali Dal, meanwhile, is yielding diminishing returns to a
movement that has dominated Punjab politics for decades.
Dynastic succession is a reality in Indian politics except for parties like BJP and fading CPM, CPI – though these
too periodically make allowances for scions proving their mettle. That none of BJP’s top rung leaders are dynasts
allows Prime Minister Narendra Modi to go on the offensive against big political families like Gandhi’s. With
umpteen regional parties originally ranged against Congress now confronting BJP, the latter is sharpening the
“parivarvaad” attack against dynasties of satraps like M Karunanidhi, Bal Thackeray, Mulayam Singh, Lalu
Prasad and lately Mamata Banerjee.
With one plank of the political attacks against him centred on “family rule”, Akhilesh has paid a backhanded
compliment to BJP for ridding SP of his family members. But political elites down to the district level are actively
invested in prolonging their own fiefdoms. Democracy in India has retained many of the dynastic and feudal
characteristics of the medieval political economy that preceded it. With personality cults flourishing, creating a
line of succession for the immediate family was a primary instinct for those who seized outsized influence in
political parties at the national, state and local levels.
But there seems to be recognition in the political class that popular acceptance for nepotism cannot be pushed
beyond a point. Yadav family politicians are conspicuously missing at Akhilesh’s side in these elections. Badaun
MP Sanghamitra Maurya hasn’t betrayed her 2019 mandate by crossing over to SP, even though her father and
senior OBC leader SP Maurya has. In Punjab and Uttarakhand, Congress is attempting a one family, one ticket
rule – discomfiting the likes of Charanjit Singh Channi and Harish Rawat. BJP has likewise denied the
prestigious Panaji seat to Manohar Parrikar’s son. Indian politics and its economy are in dire need of reforms
that improve opportunities for the youth. Dynastic politics is one of the ways those doors just don’t open wide
enough.
106. Which option best represents the main idea of the passage?
(a) Succession in dynastic politics in India is an ominous sign for many political parties.
(b) Dynastic politics are keeping India away from becoming a world leader.
(c) India’s dynastic politics is losing its charm as some cracks are showing in India’s dynastic politics .
(d) The BJP should be awarded for not being a dynastic political party.
107. It can be inferred from the passage that:
(a) The BJP is the only party with little to no practice of dynastic succession.
(b) Making allowances for scions is not a move away from nepotism.
(c) Mamata Banerjee is one of the front runners in encouraging dynastic politics.
(d) Making allowances for sons of political leaders is a move away from nepotism.
108. Out of the four marked statements, which one of the following is the odd one out?
(a) Fortunes of many political families are on the line in the upcoming five state assembly elections and for a
change these elections are seeing some family members on opposing sides of the political divide.
(b) But there seems to be recognition in the political class that popular acceptance for nepotism cannot be pushed
beyond a point.
(c) Yadav family politicians are missing at Akhilesh’s side in these elections.
(d) Democracy in India has retained many of the dynastic and feudal characteristics of the medieval political
economy that preceded it.

Head Office: 127, Zone II, MP Nagar, Bhopal |+91-7676564400| https://www.toprankers.com Page 27 of 36
109. Which of the following would present a clear challenge to the claims of the author? Consider the following
statements to be true.
I. The present Home Minister is from the CPI, whose father was also a political leader.
II. The present party President of the BJP was given the position on merit, even though his father was a Member
of Parliament.
(a) Only I (b) Only II (c) Both I and II (d) Neither I nor II.

110. Which option most accurately and correctly represents the author’s opinion?
(a) It becomes virtually impossible to rule out the competence of dynastic parties, once it is in power.
(b) The present practice of dynastic politics has its roots in the history of India.
(c) The CPM is on a declining curve and there are no hopes for it to be back in contention.
(d) The Samajwadi Party has paid the heftiest price for being a party based on dynasties.

Passage (Q.111-Q.115): NHS waiting lists are out of control. Around 7.5 million people are queueing for
hospital treatment in the UK, around 6 million of them in England. There are differences in the policies adopted
by the devolved administrations to reduce backlogs. In England, it is clear that an increased role for the private
sector is the government’s plan.

The details of the recovery plan promised by ministers are still being finalised, with NHS bosses resisting what
they regard as unrealistic targets. Finding a way to increase surgical capacity, so that more operations can happen
more quickly, is the priority. The impact of delays is both social as well as economic. But there are worrying
signs that any new contracts will tip the system’s overall balance further in favour of for-profit providers, and
away from the NHS.

The unprecedented pressure that the NHS is under ought to lead the government to invest in it, not compel it to
rely on businesses to do its work. Yet as with education, where the government chose to buy a package of
pandemic catch-up tuition from outside partners rather than funding schools themselves, ministers are opting to
meet the additional health needs of the population through the private sector.

The role of the private sector in carrying out operations on behalf of NHS England is well established. Labour’s
health spokesman, Wes Streeting, has said he would use private firms to reduce waiting lists. Outsourcing also
happens in Northern Ireland, Scotland and Wales, although to a lesser extent. But by forcing NHS England to
continue to rely on private sector backup, while refusing to commit to long-term workforce planning as
advocated by a former health secretary, the government is deliberately expanding the role of the market.
Additional NHS funding from higher national insurance should help ease pressures. But the problems in some
areas are chronic, long predating the pandemic, and a worsening staffing crisis in the social care sector is already
having knock-on effects.

Of course, ministers continue to declare their support for the NHS, which they know is valued highly by a public
that sees clearly how much worse people fare under for-profit healthcare systems such as in the US. But their
actions in beefing up the role of private providers, while refusing to take the steps that would help to secure the
NHS’s long-term future, speak louder than their words.

111. The claim made in which of the following statements is being strengthened by this piece of evidence? “People
waiting for cataracts can’t see properly; people who need joint replacements may struggle to walk.”
(a) The role of the private sector in carrying out operations on behalf of NHS England is well established.
(b) The impact of delays is both social and taxing on people’s health.
(c) There are differences in the policies adopted by the devolved administrations to reduce backlogs.
(d) Finding a way to increase surgical capacity, so that more operations can happen more quickly, is the priority.

Head Office: 127, Zone II, MP Nagar, Bhopal |+91-7676564400| https://www.toprankers.com Page 28 of 36
112. If the information given in the passage is true, then which of the following can be inferred from the passage?
(a) There is nothing wrong with the NHS getting more than just backup support from private players.
(b) There is a difference between what ministers say about the NHS and what they do for it.
(c) The Ministers would make sure that the recovery plan of the NHS is stalled.
(d) The parliamentarians genuinely care for the NHS and want it to regain its lost glory.

113. What has the author not discussed in the passage?


(a) Stance of ministers on the NHS.
(b) Steps taken by the government to ease pressure from NHS
(c) Benefits of increased role for the private sector in NHS
(d) Suggestions that could ease pressure from NHS

114. Which of the following can be attributed to be the opinion of the author?
(a) Private players would better manage the workload which has been assigned to the NHS.
(b) For the NHS to get back to its previous position, it is important that the support of private players is
completely removed.
(c) Private healthcare system such as that in the US is well suited for the public of the UK.
(d) None of the above

115. ‘The details of the recovery plan promised by ministers are still being finalised, with NHS bosses resisting what
they regard as unrealistic targets. Which of the following is the assumption behind the passage?
(a) The NHS authorities do not agree with the recovery plans.
(b) The discrepancy in the recovery plan will delay the reducing of the backlog of cases.
(c) There is a discrepancy in how the recovery plan is being perceived by ministers and NHS bosses.
(d) The recovery plan of NHS is untenable.

Passage (Q.116-Q.120): Days after meeting Prime Minister Narendra Modi in Delhi, Russian President
Vladimir Putin held a summit via video conference with Chinese President Xi Jinping. While the two meetings
may have focused on bilateral issues, the conversations appear to have overlapped in unusual ways. According
to a senior Kremlin official, after discussing with Mr. Modi India’s problems with Chinese aggression, which
were raised publicly during the visit by Defence Minister Rajnath Singh, Mr. Putin “briefed” Mr. Xi on his talks
in Delhi. The official then indicated that a trilateral summit of the leaders of Russia, India and China (RIC) could
be held in the near future, which would pick up on the Modi-Putin-Xi conversation during an RIC summit on
the sidelines of the Osaka G-20 summit in 2019. However, much has occurred between that summit and today.
Since April 2020, the two leaders have not spoken directly once, and while they have attended the same
multilateral summits (BRICS, SCO, G-20, etc.), it would be hard to see them engaging in a face-to-face format.

While India-Russia defence and bilateral ties have considerably strengthened, especially after the Modi
government’s decision to go ahead with its purchase of the S-400 missile defence systems despite the U.S. threat
of sanctions, New Delhi must tread cautiously in its trilateral and multilateral cooperation with Moscow and any
hint that Russia could play a facilitator of talks with China must be scrutinised closely. Russia and China have
consolidated their support for each other in the face of U.S. concerns over Russian action against Ukraine and
Chinese action on Taiwan. While India must continue to walk the tightrope between Moscow and Washington,
and its partners in Eurasia versus those in the Indo-Pacific, it needs to disentangle these threads from the very
potent threat it faces directly and bilaterally from its northern neighbour, where it has little choice but to follow
an independent path.

Head Office: 127, Zone II, MP Nagar, Bhopal |+91-7676564400| https://www.toprankers.com Page 29 of 36
116. Which of the following best represents the central point of this passage?
(a) Russia and India should continue to work together and solve their bilateral issues.
(b) India’s relation with Russia is similar to that friend who always stands by your side when in need.
(c) Any offer from Russia to become a facilitator between India and China should be scrutinised closely by New
Delhi.
(d) Ties between India and China have reached a new low in the last two years, which is beyond redemption.

117. With which of the following would the author be most likely to agree with?
(a) All is not well with the ties between Russia and the US.
(b) There are a lot of cultural similarities between India and Russia.
(c) Bilateral relations between different entities must be kept separate.
(d) Russia should not have brought up India while engaging with China.

118. Which of the following pieces of evidence lend support to the author’s arguments about the ties between India
and China?
(a) There was a major border conflict between the soldiers of India and China in 2020.
(b) In 2018, a Chinese platoon held 20 Indian army soldiers and released them only after much trouble.
(c) China had reserved its opinion on the recent $4 Billion deal between India and Russia.
(d) China has lent support to Russia in matters of Ukraine.

119. Which of the following may be deduced, based only on the passage?
(a) The geographical location of India is south of China.
(b) The main subject matter of Russia and China in the recent meeting was India.
(c) Mr. Xi and Mr. Modi would not engage in a face-to-face meeting again.
(d) New Delhi does not care one bit about the sanctions imposed by the US.

120. Which of the following would be helpful to know to ascertain whether Russia should become a facilitator for
Delhi to engage with China?
(a) China has invested $400 Billion in Russia; Russia’s GDP is $1 Trillion.
(b) Russia has equal and good bilateral relations with India and China.
(c) The ties between India and America are at an all-time low.
(d) Both a and b are correct.

Passage (Q.121-Q.125): The passage given below is followed by a set of questions. Choose the most appropriate
answer to each question.
From the philosophical point of view, the most brilliant results of the new method are the exact theories which
we have been able to form about infinity and continuity. We know that when we have to do with infinite
collections, for example, the collection of finite integer numbers, it is possible to establish a one-to-one
correspondence between the whole collection and a part of itself. For example, there is such a correspondence
between the finite integers and the even numbers, since the relation of a finite number to its double is one-to-
one. Thus, it is evident that the number of an infinite collection is equal to the number of a part of this collection.
It was formerly believed that this was a contradiction: even Leibnitz, although he was a partisan of the actual
infinite, denied infinite number because of this supposed contradiction. But to demonstrate that there is a
contradiction, we must suppose that all numbers obey mathematical induction. To explain mathematical
induction, let us call by the name “hereditary property’ of a number, a property which belongs to n + 1 whenever
it belongs to n. Such is, for example, the property of being greater than 100. If a number is greater than 100, the
next number after it is greater than 100. Let us call by the name “inductive property” of a number a hereditary
property which is possessed by the number zero. Such a property must belong to 1, since it is hereditary and
belongs to 0; in the same way, it must belong to 2, since it belongs to 1; and so on. Consequently, the numbers
of daily life possess every inductive property. Now, amongst the inductive properties of numbers is found the
Head Office: 127, Zone II, MP Nagar, Bhopal |+91-7676564400| https://www.toprankers.com Page 30 of 36
following. If any collection has the number n, no part of this collection can have the same number n.
Consequently, if all numbers possess all inductive properties, there is a contradiction with the result that there
are collections which have the same number as a part of themselves. This contradiction, however, ceases to
subsist as soon as we admit that there are numbers which do not possess all inductive properties. And then it
appears that there is no contradiction in infinite number. Cantor has even created a whole arithmetic of infinite
numbers, and by means of this arithmetic, he has completely resolved the former problems on the nature of the
infinite which have disturbed philosophy since ancient times.

The problems of the continuum are closely connected with the problems of the infinite and their solution is
effected by the same-means. The paradoxes of Zeno the Eleatic and the difficulties in the analysis of space, of
time, and of motion, are all completely explained by means of the modern theory of continuity. This is because
a non-contradictory theory has been found, according to which the continuum is composed of an infinity of
distinct elements; and this formerly appeared impossible. The elements cannot all be reached by continual
dichotomy; but it does not follow that these elements do not exist.

From this follows a complete revolution in the philosophy of space and time. The realist theories which were
believed to be contradictory are so no longer, and the idealist theories have lost any excuse there might have
been for their existence. The flux, which was believed to be incapable of analysis into indivisible elements,
shows itself to be capable of mathematical analysis, and our reason shows itself to be capable of giving an
explanation of the physical world and of the sensible world without supposing jumps where there is continuity,
and also without giving up the analysis into separate and indivisible elements.

121. What is the way in which the author of the above passage presents his views?
(a) Underlines the advantages of a new method, explains them, and glorifies the method.
(b) Introduces a new method, explains it, relates its usefulness and then exemplifies it.
(c) Disagrees with an existing norm, states ways of changing, and typifies it.
(d) Counters a hypothesis, exposes its loopholes and revokes it.

122. According to the passage, in what way has the problem of the infinite number been resolved?
(a) By stating that the property that the number of an infinite collection is equal to the number of a part of this
collection, is a contradiction.
(b) By understanding that the property that the number of an infinite collection is equal to the number of a part
of this collection, is no more supposed to be a contradiction.
(c) With possible solutions, the property that the number of an infinite collection is equal to the number of a part
of this collection, can erase a contradiction.
(d) The contradiction has finally become an established property.

123. Extending the author’s logic to different situations, which of these is he most likely to agree with?
(a) The remedy for all diseases is possible to develop as there is an acceptance of its presence in the subconscious
mind.
(b) The subconscious mind cannot be formed solely of electric signals. The emotions must play a role in it.
(c) “Emotions are immeasurable” is a statement that need be revoked in order that the human mind pursue its
goal of learning.
(d) Learning about learning is an odious task and one in which there should be no investing of time.

124. According to the passage, what is the relation between the inductive property and the hereditary property?
(a) An inductive property is an example of the hereditary property.
(b) An inductive property is an extension of the hereditary property in a totally unknown field.
(c) A hereditary property logically follows an inductive property.
(d) An inductive property is an extension of the hereditary property in a known field.

Head Office: 127, Zone II, MP Nagar, Bhopal |+91-7676564400| https://www.toprankers.com Page 31 of 36
125. The supposed contradiction that is stated in the passage related to the infinite number is that
(a) the infinite number cannot be a part of the whole and, hence, cannot be realized.
(b) the infinite number can be a part of itself and, hence, cannot be realized.
(c) all numbers obey mathematical induction.
(d) all the numbers do not obey mathematical induction.

Passage (Q.126-Q.130): Religious leaders are supposed to be detached and ‘liberated’ from the affairs of the
world, but the manner in which dozens of Lingayat seers have openly dabbled in politics by lobbying for the
continuation of B S Yediyurappa as Karnataka Chief Minister is an indication of how far they have strayed from
the path of renunciation. The doctrine of separation of Church and State calls for a distance between the political
apparatus and religious organisations, but this line has been breached all too often. The political clout that these
men in ochre robes wield has never been in doubt, but to dictate to a political party that a Chief Minister should
not be removed only because he belongs to their community does not auger well for democracy and secularism.
It is a different matter though that, irrespective of the party, it is usually an undemocratic ‘high command’ that
has the final say in the choice of the Chief Minister, and not the MLAs as mandated by the Constitution.
Nonetheless, religious and spiritual leaders have no business to play the role of power brokers.
While all political parties are guilty of pandering to various mutts and their heads in view of their supposed
influence over their followers, the BJP has taken it to an altogether different level in its bid to shred secularism,
a basic requirement of democracy, and wield its Hindutva ideology to gain political dominance. It has come back
to bite the party, as the seers threaten to derail the BJP’s electoral prospects if it dares replace Yediyurappa. The
BJP has already had a taste of it during the 2013 Assembly elections, when the seers threw their weight behind
Yediyurappa who had floated his own outfit, the Karnataka Janata Party, leading to the BJP’s defeat in many
seats. It is no secret that many religious leaders interfere in the government’s functioning and exert pressure on
the Chief Minister to induct their favourites as ministers and officers in plum posts. But what is most unfortunate
is that the effect of what the seers have said openly has the effect of dividing people on the basis of caste and
community.
If the seers are keen on a life of politics, they should drop the pretence of renunciation and contest elections, as
Yogi Adityanath and others have done. People look up to seers and saints with great reverence. They bring no
honour to themselves by indulging in political manoeuvres. Politics and religion will both retain legitimacy only
so long as they remain separate.

126. What is the central theme of the passage?


(a) Religious leaders are supposed to be detached from the affairs of the world.
(b) Chief Minister should not be removed only because he belongs to their community.
(c) Seers had always been and will remain in politics.
(d) The intervention of the religious leaders in politics does not bode well for democracy.

127. The doctrine of separation of Church and State calls for a distance between
(a) Non- political and religious organisations.
(b) Political apparatus and religious organisations.
(c) Various political apparatus.
(d) Religious organisations.

128. All of the following can be inferred except


(a) The case of removing B S Yediyurappa as Karnataka Chief Minister is not unprecedented.
(b) No political party in India is untouched to playing up to the religious sentiments.
(c) There are incidents of religious seers who joined politics after renunciation of religion sentiments.
(d) Religious leaders, at times, are the backbones behind a prominent leader.

Head Office: 127, Zone II, MP Nagar, Bhopal |+91-7676564400| https://www.toprankers.com Page 32 of 36
129. Directions: Each question has a set of sequentially ordered statements. Each statement can be classified as one
of the following.
Facts, which deal with pieces of information that one has heard, seen or read, and which are open to discovery
or verification (the answer option indicates such a statement with an ‘F’)
Inferences, which are conclusions drawn about the unknown, on the basis of the known (the answer option
indicates such a statement with an ‘I’).
Judgments, which are opinions that imply approval or disapproval of persons, objects, situations and occurrences
in the past, the present or the future (the answer option indicates such a statement with a ‘J’).
I. The doctrine of separation of Church and State calls for a distance between the political apparatus and
religious organisations, but this line has been breached all too often.
II. Nonetheless, religious and spiritual leaders have no business to play the role of power brokers.
III. During the 2013 Assembly elections, the seers threw their weight behind Yeddyurappa who had floated his
own outfit, the Karnataka Janata Party, leading to the BJP’s defeat in many seats.
IV. It is no secret that many religious leaders interfere in the government’s functioning and exert pressure on the
Chief Minister to induct their favourites as ministers and officers in plum posts.
V. People look up to seers and saints with great reverence.
(a). FJFFJ (b). JJIFI (c). FIIFJ (d). JIFFJ

130. ‘Politics and religion will both retain legitimacy only so long as they remain separate.’ Which of the following
is the assumption behind the passage?
(a) Politics and religion are two sides of the same coin.
(b) Religion is drops of oil in political water
(c) Politics and Religion can retain legitimacy if only they remain separate from each other.
(d) Religion and politics are parallel lines.

Passage (Q.130-Q.135): 2022 offers a hint of hope after a year of gloom. Vaccines are rolling out, and we might
permit ourselves visions of normality as the battle against the COVID-19 pandemic at last swings in our favour.
While we may soon prevail against COVID-19, we can’t ignore an increasing problem that the fight against the
virus has worsened. Plastics have been deployed in great quantities as a shield against COVID. But little attention
has been paid to where the increased plastic waste will end up. The sad irony is we were on the cusp of real
victories against plastic pollution just as the coronavirus pandemic began.
In 2019, Prime Minister Narendra Modi committed to completely phase out single-use plastics by 2022. The
commitment called for better arrangements to collect, store, and recycle single-use plastic. The movement was
also international. The UN Environment Programme, with the support of Norway and Japan, undertook a
multiyear assessment of how plastic finds its way into riverways, and ultimately the ocean, through projects like
CounterMEASURE. And National Geographic’s “Sea to Source: Ganges” Expedition brought together four
countries, including India and Bangladesh, to holistically study plastic pollution within the Ganges River basin.
The pandemic halted and, in some cases, reversed much of this progress. Plastics, especially single-use plastics,
became more ubiquitous. Masks, sanitiser bottles, personal protective equipment, food packaging, water bottles:
Life came to be ensconced in a plastic shell.
In time, this plastic will disintegrate into tiny particles of less than five millimetres — known as microplastics
— and move through water bodies and farm soil to enter the food we eat and the air we breathe. We know that
only 9 per cent of all plastic ever produced has been recycled, while 79 per cent of all plastic produced can be
found in the world’s landfills and in our air, water, soil, and other natural systems. Plastic doesn’t belong in our
bodies and it doesn’t belong in nature. But plastic is still important. Its central role in durable goods, medicine
and food safety means that it is not practical to get rid of entirely. Instead, we must be more thoughtful about
where, when and how we use it. We need an approach that includes reducing the manufacture of new fossil fuel-
based plastics, improving waste collection and disposal, and developing and using alternatives.

Head Office: 127, Zone II, MP Nagar, Bhopal |+91-7676564400| https://www.toprankers.com Page 33 of 36
131. Which one of the following is an assumption that the author relies upon in the passage?
(a) Plastic waste disposal is the next bigger challenge the government takes on after the subjugation of COVID-
19.
(b) Tackling Plastic waste is as challenging as dealing with Covid-19.
(c) With the increase of plastic use during the Covid times, the plastic waste disposal will emerge as a major
environmental challenge for the government.
(d) There is a cure for Covid-19 but there is no cure for perils of plastic usage.

132. ‘But plastic is still important. Its central role in durable goods, medicine and food safety means that it is not
practical to get rid of entirely.’ Which one of the following conclusions can be correctly drawn from the
statements above?
I. One can never get rid of use of plastics.
II. Plastic is here to stay till an alternative is found.
III. The use of plastic will prevail for certain commodities for a very long time.
(a) Only I (b) Only II (c) II and III (d) I and III.

133. ‘The commitment called for better arrangements to collect, store, and recycle single-use plastic.’ Which of the
following statement/(s) from the passage itself/themselves belies the above statement?
I. We know that only 9 per cent of all plastic ever produced has been recycled, while 79 per cent of all plastic
produced can be found in the world’s landfills and in our air, water, soil, and other natural systems.
II. Plastic doesn’t belong in our bodies and it doesn’t belong in nature.
III. Life came to be ensconced in a plastic shell.
(a) Only I (b) Only II (c) I and II (d) I, II & III.

134. Life came to be ensconced in a plastic shell. What can be inferred from the given statement?
(a) Life revolved around plastics with the coming of Pandemic.
(b) Plastic use became a way of life during pandemic.
(c) People realized more and more the importance of plastic during pandemic.
(d) The use of plastics declined during pandemic.

135. Which of the following will the author least likely to agree with
(a) There is an urgent need to consolidate efforts nationally and internationally to address the mayhem of use of
plastics.
(b) To work towards finding an alternative organic source to stop the use of plastics.
(c) To start various programmes for public awareness and appeal to the minimum use of plastics.
(d) To bring a draconian law to deal with the inordinate use of plastics.

Head Office: 127, Zone II, MP Nagar, Bhopal |+91-7676564400| https://www.toprankers.com Page 34 of 36
SECTION - E: QUANTITATIVE TECHNIQUES

Directions (Q.136-Q.140): Study the paragraph and answer the questions that follow. In a senior secondary
school, every class is divided into six sections A, B, C, D, E and F. There are 50 students in each section of the
class. In the sections A, B and C of class 10th, the girl to boy ratio is 2 : 3, while in sections D and E, the girl to
boy ratio is 3 : 7. On the other hand, there are equal number of boys and girls in the section F of class 10th. The
number of boys in the sections A and B of class 12th is 80% and 90% of the number of boys in the sections A
and B of class 10th. While, the number of girls in the other sections of class 12th is 60% of the number of boys
in the respective sections of the class 10th.

136. What is the boy to girl ratio in the section F of class 12th?
(a) 3 : 2 (b) 4 : 1 (c) 5 : 3 (d) 7:3

137. How many total number of boys are there in the sections D and E of class 12th?
(a) 21 (b) 58 (c) 35 (d) 42

138. How many girls are there in the section C of class 12th?
(a) 12 (b) 15 (c) 18 (d) 20

139. How many boys are there in the section A of class 12th?
(a) 20 (b) 24 (c) 28 (d) 30

140. How many total number of girls are there in the sections A. B and C of class 10th?
(a) 30 (b) 45 (c) 60 (d) 75

Direction (Q.141-Q.145): Study the given information carefully and answer the following questions
accordingly.
In a hospital, there are 4 wards. In ward A. 56 patients are admitted, and the ratio of male to female is 3 : 4. In
ward B, 60 patients are admitted and the ratio of male to female is 3 : 1. In ward - C, 52 patients are admitted
and the ratio of male to female is 8 : 5. In ward - D, 44 patients are admitted and the ratio of male to female is
6 : 5.

141. Find the total number of females patients admitted in ward –A and Ward - C together.
(a) 32 (b) 42 (c) 68 (d) 52

142. The total number of male patients in ward B and ward D together are approximately what percent of the total
number of patients in the same wards together?
(a) 66% (b) 56% (c) 72% (d) 52%

143. Find the average number of patients in all the wards.


(a) 50 (b) 52 (c) 53 (d) 55

144. Find the ratio of total number of male patients admitted in ward –A to Ward - C .
(a) 1:4 (b) 2:3 (c) 3:4 (d) 3:5

145. Total number of patients in all the wards is what percentage more than the total number of all male patients.
(a) 50% (b) 69.6% (c) 59.6% (d) 79.6%

Head Office: 127, Zone II, MP Nagar, Bhopal |+91-7676564400| https://www.toprankers.com Page 35 of 36
Directions (Q.146-Q.150): Study the following pie chart and table carefully and answer the following
questions.
Pie chart shows percentage distribution of total number of students in 5 different games (Cricket, Football,
Badminton, Hockey and Volleyball) in 2018 and table shows the ratio of boys to girls in these 5 games in 2018
and in 2019.

Total no. of students in 2018 = 20,000

Cricket
Volleyball
20%
25%

Hockey
10% Football
30%
Badminton
15%

Cricket Football Badminton


Hockey Volleyball

146. Total number of boys in Cricket, Football & Hockey together in 2018 is what percent of total number of
students in 2018?
(a) 33% (b) 35% (c) 23% (d) 32%

147. What is the ratio of total number of girls in cricket, football and badminton together in 2018 to the boys in
Volleyball and Badminton together in 2018?
(a) 54 : 11 (b) 13 : 52 (c) 13 : 54 (d) 35 : 13

148. If number of boys in Football in 2019 is increased by 50% as compared to previous year and number of boys in
Cricket in 2019 is increased by 20% as compared to previous year, then what is the total number of girls in
Football in 2019 and number of girls in Cricket in 2019 together?
(a) 5166 (b) 5058 (c) 5194 (d) 5108

149. If total student in 2019 in Hockey is 5000 and no. of boys in Hockey is increased by 20% from 2018 to 2019,
then find value of x?
(a) 36 (b) 45 (c) 38 (d) 49
3
150. If total students in Hockey in 2019 are 4000 and no. of girls in Hockey is increased by 233 4 % from 2018 to
2019, then find number of boys in Hockey in 2019.
(a) 1580 (b) 1330 (c) 1470 (d) 1390

Head Office: 127, Zone II, MP Nagar, Bhopal |+91-7676564400| https://www.toprankers.com Page 36 of 36

You might also like